Quaternion Multiplication: Expanding and Simplifying

Click For Summary
SUMMARY

The discussion focuses on the expansion and simplification of the product of two quaternions: (3 + 2i + 3j + 4k)(3 + 3i + 2j + 5k). The participant successfully expanded the brackets while maintaining the order of the imaginary units, acknowledging that quaternion multiplication is not commutative. The justification for the method lies in the distributive property of quaternion multiplication over addition, as defined in quaternion algebra.

PREREQUISITES
  • Understanding of quaternion algebra
  • Familiarity with the properties of imaginary units i, j, and k
  • Knowledge of the distributive property in algebra
  • Basic skills in expanding polynomial expressions
NEXT STEPS
  • Study quaternion multiplication properties in detail
  • Learn about the non-commutative nature of quaternion operations
  • Explore applications of quaternions in 3D graphics and physics
  • Practice additional quaternion expansion problems for mastery
USEFUL FOR

Students studying advanced mathematics, particularly those focusing on algebra and quaternion theory, as well as professionals in computer graphics and physics who utilize quaternion mathematics.

flash
Messages
66
Reaction score
0

Homework Statement


Expand and simplify the product of two quaternions:

(3 + 2i + 3j + 4k)(3 + 3i + 2j + 5k)

Justify your response.


The Attempt at a Solution



I have done this by expanding brackets normally, keeping the ijk's in the same order because the multiplication is not commutative. I have the correct answer, I just can't think how to justify it. Why should expanding brackets necessarily work?
 
Physics news on Phys.org
flash said:
Why should expanding brackets necessarily work?

Isn't that how multiplication of the quaternions is defined?
 
Last edited:
flash said:
Why should expanding brackets necessarily work?

It works necessarily because quaternion multiplication distributes over quaternion addition per the definitions of quaternion multiplication and addition.
 
Thanks guys, that's all I needed.:cool:
 
Question: A clock's minute hand has length 4 and its hour hand has length 3. What is the distance between the tips at the moment when it is increasing most rapidly?(Putnam Exam Question) Answer: Making assumption that both the hands moves at constant angular velocities, the answer is ## \sqrt{7} .## But don't you think this assumption is somewhat doubtful and wrong?

Similar threads

  • · Replies 2 ·
Replies
2
Views
2K
  • · Replies 1 ·
Replies
1
Views
1K
  • · Replies 2 ·
Replies
2
Views
1K
  • · Replies 11 ·
Replies
11
Views
2K
  • · Replies 1 ·
Replies
1
Views
2K
  • · Replies 5 ·
Replies
5
Views
3K
  • · Replies 2 ·
Replies
2
Views
7K
  • · Replies 7 ·
Replies
7
Views
3K
  • · Replies 2 ·
Replies
2
Views
2K
  • · Replies 1 ·
Replies
1
Views
2K